3
$\begingroup$

A price is a number $ \frac{x}{100}$ with a positive integer $x$. How many distinct prices can there be such that their product equals their sum, i.e., what is the largest $n$ such that positive integers $x_1 < \cdots < x_n$ exist with $x_1 \cdots x_n = 100^{n-1} \cdot(x_1 + \cdots + x_n)$?

As noted in OEIS sequence A382547, it is easy to see that $n \le 274$, but this is probably much too large an upper bound. Can anyone find a smaller bound?

$\endgroup$
3
  • $\begingroup$ Related: math.stackexchange.com/questions/2919430/sum-equals-the-product $\endgroup$ Commented May 26 at 15:23
  • $\begingroup$ The OEIS entry links to lots of additional information, but just to put this here: the requirement that the prices are distinct is essential. Otherwise there’s no upper bound. $\endgroup$ Commented May 26 at 18:10
  • 1
    $\begingroup$ Yes, dropping the distinctness requirement gives the infinite sequence A380887. For example, for each $n$ the numbers $1, \dots, 1, k+1, k*(k+n-1)$ with $n-2$ times the number $1$ and $k := 100^{n-1}$ are a solution. $\endgroup$ Commented May 27 at 5:04

2 Answers 2

2
$\begingroup$

Let $n$ be a positive integer, and let $x_1<\ldots<x_n$ be positive integers such that $$\prod_{k=1}^nx_k=100^{n-1}\sum_{k=1}^nx_k.\tag{1}$$ As noted in the comments under OEIS sequence A382547, for all $k$ you have $x_k\geq k$, and so $$\prod_{k=1}^nx_k\geq x_n\prod_{k=1}^{n-1}k=(n-1)!x_n.\tag{2}$$ Also for all $k$ you have $x_k\leq x_n$, and so $$100^{n-1}\sum_{k=1}^nx_k\leq100^{n-1}\sum_{k=1}^nx_n=100^{n-1}nx_n.\tag{3} $$ Substituting this into equation $(1)$, it follows that $$100^{n-1}nx_n\geq (n-1)!x_n,$$ and hence that $100^{n-1}n\geq(n-1)!$. This in turn implies that $n\leq274$.

We can expand the upper bound $(3)$ a bit to get $$100^{n-1}(n-1)x_{n-1}+100^{n-1}x_n\geq(n-1)!x_n,$$ from which it follows that $$\frac{x_{n-1}}{x_n}\geq\frac{(n-2)!}{100^{n-1}}-\frac{1}{n-1}.$$ If $n=274$ then this implies $x_{n-1}>0.4874x_n$. Then similar to $(2)$ we find $$\prod_{k=1}^nx_k>0.4874x_n^2\prod_{k=1}^{n-2}k=0.4874(n-2)!x_n^2,$$ and so as before $$100^{n-1}nx_n>0.4874(n-2)!x_n^2,$$ from which it follows that $$x_n<\frac{100^{n-1}n}{0.4874(n-2)!}<1145.$$ This gives us an upper bound on the number of factors of $5$ on the left hand side of $(1)$: We have $$v_5\left(\prod_{k=1}^nx_k\right)\leq v_5(x_n!)\leq v_5(1144!)=283.$$ But for $n=274$ we see on the right hand side of $(1)$ that $$v_5\left(100^{n-1}\sum_{k=1}^nx_k\right)=(n-1)v_5(100)+v_5\left(\sum_{k=1}^nx_k\right)\geq(n-1)v_5(100)=273\times2=546,$$ a contradiction. We conclude that $n\leq273$.

$\endgroup$
1
$\begingroup$

For positive integers $m$ and $v$, let $$b_5(m,v) := \min \{ a_m : a \in \mathbb N^m, 1 \le a_1 < \cdots < a_m, \ v_5(a_1 \cdot\ldots\cdot a_m) \ge v \}.$$ The values of $b_5(m,v)$ that we will use in the following can easily be determined by computer.

Let $n \ge 2$ be an integer and $x \in \mathbb N^n$ a solution, i.e. $1 \le x_1 < \cdots < x_n$ with $$x_1 \cdot\ldots\cdot x_n = 100^{n-1} (x_1 + \cdots + x_n),$$ which implies $x_n \ge b_5(n, 2n-2)$.

Following Servaes' idea, we estimate $$\begin{eqnarray*} (n-1)! \cdot x_n &\le& x_1 \cdot\ldots\cdot x_n\\ &=& 100^{n-1}(x_1 + \cdots + x_n)\\ &\le& 100^{n-1}(n-1)x_{n-1} + 100^{n-1}x_n \end{eqnarray*}$$ and get $$x_{n-1} \ge \alpha_n \, x_n \quad \mbox{with} \quad \alpha_n := \frac{(n-2)!}{100^{n-1}} - \frac1{n-1}.$$ In the following, let $n \ge 270$, which ensures $\alpha_n > 0$. Then we have: $$\begin{eqnarray*} (n-2)! \cdot x_{n-1} \cdot x_n &\le& x_1 \cdot\ldots\cdot x_n\\ &=& 100^{n-1}(x_1 + \cdots + x_n)\\ &\le& 100^{n-1} n x_n \end{eqnarray*}$$ $$(n-2)! \cdot \alpha_n \, x_n \le (n-2)! \cdot x_{n-1} \le 100^{n-1} n$$ $$x_n \le \frac{100^{n-1}n}{(n-2)! \cdot \alpha_n} =: \beta_n \tag{1}$$ Furthermore we have: $$\begin{eqnarray} (n-2)! \cdot x_{n-1} \cdot (x_{n-1} + 1) &\le& x_1 \cdot\ldots\cdot x_n\\ &=& 100^{n-1} (x_1 + \cdots + x_n)\\ &\le& 100^{n-1} (n-1) x_{n-1} + 100^{n-1} \cdot \beta_n \end{eqnarray} \tag{2}$$ $$\begin{eqnarray} (n-3)! \cdot x_{n-2} (x_{n-2} + 1) (x_{n-2} + 2) &\le& x_1 \cdot\ldots\cdot x_n\\ &=& 100^{n-1}(x_1 + \cdots + x_n)\\ &\le& 100^{n-1} (n-2) x_{n-2} + 100^{n-1} \cdot 2 \beta_n \end{eqnarray} \tag{3}$$ $$\begin{eqnarray} (n-4)! \cdot x_{n-3} (x_{n-3} + 1) (x_{n-3} + 2) (x_{n-3} + 3) &\le& x_1 \cdot\ldots\cdot x_n\\ &=& 100^{n-1}(x_1 + \cdots + x_n)\\ &\le& 100^{n-1} (n-3) x_{n-3} + 100^{n-1} \cdot 3 \beta_n \end{eqnarray} \tag{4}$$ In the case of $n = 274$, $x_n \ge b_5(274, 546) = 5500$. But (1) gives $x_n \le 1144$, a contradiction.

In the case of $n = 273$, (1) reads $x_n \le 8549$, so $v_5(x_n) \le 5$ and $$544 = 2n-2 \le v_5(x_1 \cdot\ldots\cdot x_n) \le v_5(x_1 \cdot\ldots\cdot x_{n-1}) + 5,$$ thus $v_5(x_1 \cdot\ldots\cdot x_{n-1}) \ge 539$, so $x_{n-1} \ge b_5(272,539) = 5375$. But (2) gives $x_{n-1} \le 1536$, a contradiction.

In the case of $n = 272$, (1) reads $x_n \le 64877$, so $v_5(x_n) \le 6$ and $$542 = 2n-2 \le v_5(x_1 \cdot\ldots\cdot x_n) \le v_5(x_1 \cdot\ldots\cdot x_{n-1}) + 6,$$ thus $v_5(x_1 \cdot\ldots\cdot x_{n-1}) \ge 536$, so $x_{n-1} \ge b_5(271,536) = 5350$. But (2) gives $x_{n-1} \le 4293$, a contradiction.

In the case of $n = 271$, (1) reads $x_n \le 523728$, so $v_5(x_n) \le 8$, and (2) yields $x_{n-1} \le 12622$, so $v_5(x_{n-1}) \le 5$ and $$540 = 2n-2 \le v_5(x_1 \cdot\ldots\cdot x_n) \le v_5(x_1 \cdot\ldots\cdot x_{n-2}) + 13,$$ thus $v_5(x_1 \cdot\ldots\cdot x_{n-2}) \ge 527$, so $x_{n-2} \ge b_5(269,527) = 5200$. But (3) gives $x_{n-2} \le 2680$, a contradiction.

In the case of $n = 270$, (1) reads $x_n \le 5395811$, so $v_5(x_n) \le 9$, (2) yields $x_{n-1} \le 43004$, so $v_5(x_{n-1}) \le 6$, and (3) yields $x_{n-2} \le 7188$, so $v_5(x_{n-2}) \le 5$ and $$538 = 2n-2 \le v_5(x_1 \cdot\ldots\cdot x_n) \le v_5(x_1 \cdot\ldots\cdot x_{n-3}) + 20,$$ thus $v_5(x_1 \cdot\ldots\cdot x_{n-3}) \ge 518$, so $x_{n-3} \ge b_5(267,518) = 5050$. But (4) gives $x_{n-3} \le 3396$, a contradiction.

Consequently, $n \le 269$.

$\endgroup$
0

You must log in to answer this question.

Start asking to get answers

Find the answer to your question by asking.

Ask question

Explore related questions

See similar questions with these tags.